Vous êtes sur la page 1sur 4

Statistical Mechanics - Homework Assignment 1

Alejandro Gomez Espinosa

February 5, 2013
Pathria 1.4 In a classical gas of hard spheres (of diameter D), the spatial distribution of the particles
is no longer uncorrelated. Roughly speaking, the presence of n particles in the system leaves only
a volume (V nv
0
) available for the (n + 1)th particle; clearly, v
0
would be proportional to D
3
.
Assuming that Nv
0
<< V , determine the dependence of (N, V, E) on V (compare to equation
(1.4.1)) and show that, as a result of this, V in the ideal-gas law (1.4.3) gets replaced by (V b),
where b is four times the actual volume occupied by the particles.
To determine the dependence of in this case, the relation must reect that each time that we
introduce a particle in the volume V we will end up with less space. This scenario can be represented
by
(N, E, V ) =
N

n=1
(V (n 1)v
0
) (1)
Now, let us use the eq. 1.4.2 to derive the dependences on V:
P
T
= k
ln
V
= k

V
ln
_
N

n=1
(V (n 1)v
0
)
_
= k

V
N

n=1
ln(V (n 1)v
0
)
= k
N

n=1
1
V (n 1)v
0
=
k
V
N

n=1
1
1
(n1)v
0
V

k
V
N

n=1
_
1 +
(n 1)v
0
V
_
=
k
V
_
N +
N(N + 1)v
0
2V

Nv
0
V
_
=
kN
V
_
1 +
(N 1)v
0
2V
_

kN
V
_
1
(N 1)v
0
2V
_
1
= kN
_
V
(N 1)v
0
2
_
1
where the term
(N1)v
0
2
is clearly the volume of the particles added to the system. Then, taken into
account the condition that v
0
D
3
we can replace in the ideal-gas law the term V with (V 4b).

gomez@physics.rutgers.edu
1
Pathria 1.7 Study the statistical mechanics of an extreme relativisitic gas characterized by the single-
particle energy states
(n
x
, n
y
, n
z
) =
hc
2L
(n
2
x
+n
2
y
+n
2
z
)
1/2
(2)
instead of (1.4.5), along the lines followed in Section 1.4. Show that the ratio C
P
/C
V
in this case
is 4/3, instead of 5/3.
Using the procedure following in Pathria (eq. 1.4.5) with (2), we can dene:
(n
x
, n
y
, n
z
)
1/2
=
2L
hc
(n
x
, n
y
, n
z
) =
4L
2

2
h
2
c
2
=
4V
2/3

2
h
2
c
2
where L
3
= V . Extending this argument for (N, E, V ), gives the relation:
3N

r=1
n
2
r
=
4V
2/3
E
2
h
2
c
2
(3)
From this expresion, we can conclude that the combination V
2/3
E
2
will enter into the expression
for . Therefore S(N, V, E) = S(N, V
2/3
E
2
), which denes a reversible adiabatic process when:
V
2/3
E
2
= const E = const V
1/3
(4)
Then, using Eq. 1.4.10:
P =
_
E
V
_
N,S
=
1
3
const V
4/3
(5)
resulting in the adiabatic relation:
PV
4/3
= const (6)
Finally, comparing to Eq. 1.4.30 for adiabatic process: PV

, where =
Cp
Cv
, we found the nal
result:
C
p
C
v
=
4
3
(7)
2
Pathria 1.8 Consider a system of quasiparticles whose energy eigenvalues are given by
(n) = nh; n = 0, 1, 2, ... (8)
Obtain an asymptotic expression for the number of this system for a given number N of the
quasiparticles and a given total energy E. Determine the temperature T of the system as a function
of E/N and h, and examine the situation for which E/(Nh) >> 1.
For a system of quasiparticles with energy (8), the total energy of the system must be the sum of
the particles in the system i:
E =

i=0
inh (9)
In addition, since the energy of the eigenvalues of the quasiparticles is quantized, we know that the
energy must be equal to E = ph, where p is the momentum of the particle. Comparing (10) with
this relation is easy to see that p =

in.
Next we have to estimate the number of states in our system. Since we know now the momentum
of the quanta, we have to distribute this momentum into N particles. Then, the number of state is
approximated to the combination of P +N states with P combinations. Hence,

(P +N)!
P!N!
ln = ln(P +N)! ln P! ln N!
(P +N) ln(P +N) (P +N) P ln P +P N ln N +N
= P ln
_
P +N
P
_
+N ln
_
P +N
N
_
Replacing P = E/h, we found the relation for the entropy:
S = k ln
=
kE
h
ln
_
1 +
Nh
E
_
+kN ln
_
E
Nh
+ 1
_
Following by the temperature:
1
T
=
_
S
E
_
N
=
k
h
ln
_
1 +
Nh
E
_

kE
h
Nh
E
2
1 +
Nh
E
+kN
1
Nh
E
Nh
+ 1
=
k
h
ln
_
1 +
Nh
E
_

kN
E +Nh
+
kN
E +Nh
T =
h
k ln
_
1 +
Nh
E
_
Finally, in the case that
Nh
E
>> 1, ln
_
1 +
Nh
E
_

Nh
E
. Therefore, the temperature could be
approximated by:
T
E
kN
(10)
3
Pathria 1.13 If the two gases considered in the mixing process of Section 1.5 were initially at dier-
ent temperatures, say T
1
and T
2
, what would the entropy of mixing be in that case? Would the
contribution arising from this cause depend on whether the two gases were dierent or identical?
In the case of two ideal gases with dierent temperature, eq. 1.5.1 becomes:
S
i
= N
i
k ln V
i
+
3
2
N
i
k
_
1 + ln
_
2m
i
kT
i
h
2
__
; i = 1, 2 (11)
After the mixing, the total entropy would be:
S
T
=
2

i=1
_
N
i
k ln V +
3
2
N
i
k
_
1 + ln
_
2m
i
kT
h
2
___
(12)
where V = V
1
+ V
2
and T is the nal temperature of the system. Therefore, using (11) and (12),
the entropy of mixing is given by
(S) = S
T

2

i=1
S
i
= N
1
k ln V +N
2
k ln V N
1
k ln V
1
N
2
k ln V
2
+
3
2
N
1
k +
3
2
N
2
k
3
2
N
1
k
3
2
N
2
k +
+
3
2
N
1
k ln
_
2km
1
T
h
2
_
+
3
2
N
2
k ln
_
2km
2
T
h
2
_

3
2
N
1
k ln
_
2km
1
T
1
h
2
_

3
2
N
2
k ln
_
2km
2
T
2
h
2
_
= N
1
k ln
_
V
V
1
_
+N
2
k ln
_
V
V
2
_
+
3
2
N
1
k ln
_
T
T
1
_
+N
2
k ln
_
T
T
2
_
= N
1
k
_
ln
_
V
1
+V
2
V
1
_
+ ln
_
T
T
1
_
3/2
_
+N
2
k
_
ln
_
V
1
+V
2
V
2
_
+ ln
_
T
T
2
_
3/2
_
Since the increase in the entropy in this case depends upon the temperature, there will be no
distinction whether the two gases were dierent or identical. This is because the temperature
measures the energy of a system, and since the nal temperature after the mixing does not depend
upon the particles are the same, the contribution cannot be atributed to the dierence of the gases.
4

Vous aimerez peut-être aussi